Calculus 8th Edition

Published by Cengage
ISBN 10: 1285740629
ISBN 13: 978-1-28574-062-1

Chapter 6 - Inverse Functions - 6.8 Indeterminate Forms and I'Hospital's Rule - 6.8 Exercises - Page 500: 48

Answer

$\infty$

Work Step by Step

Given $$\lim _{x \rightarrow \infty} x^{3 / 2} \sin (1 / x)$$ Rewrite the limit as the following \begin{aligned} \lim _{x \rightarrow \infty} x^{3 / 2} \sin (1 / x)&= \lim _{x\to \infty \:}\left(\frac{\sin \left(\frac{1}{x}\right)}{\frac{1}{x^{\frac{3}{2}}}}\right)\\ \end{aligned} Apply L'hopital's rule \begin{aligned} \lim _{x \rightarrow \infty} x^{3 / 2} \sin (1 / x)&= \lim _{x\to \infty \:}\left(\frac{\sin \left(\frac{1}{x}\right)}{\frac{1}{x^{\frac{3}{2}}}}\right)\\ &= \lim _{x\to \infty \:}\left(\frac{-\frac{\cos \left(\frac{1}{x}\right)}{x^2}}{-\frac{3}{2x^{\frac{5}{2}}}}\right)\\ &= \lim _{x\to \infty \:}\left(\frac{2}{3}x^{\frac{1}{2}}\cos \left(\frac{1}{x}\right)\right)\\ &= \frac{2}{3}\cdot \lim _{x\to \infty \:}\left(x^{\frac{1}{2}}\cos \left(\frac{1}{x}\right)\right)\\ &= \frac{2}{3}\cdot \lim _{x\to \infty \:}\left(x^{\frac{1}{2}}\right)\cdot \lim _{x\to \infty \:}\left(\cos \left(\frac{1}{x}\right)\right)\\ &= \frac{2}{3}\cdot \infty \cdot \:1\\ &=\infty \end{aligned}
Update this answer!

You can help us out by revising, improving and updating this answer.

Update this answer

After you claim an answer you’ll have 24 hours to send in a draft. An editor will review the submission and either publish your submission or provide feedback.